Você está na página 1de 19

(19) 3251-1012 O ELITE RESOLVE ITA 2013 - FSICA

1

FSICA

Se precisar, use os seguintes valores para as constantes:
carga do prton
19
1,6 10 C

= ; massa do prton
27
1,7 10 kg

= ;
acelerao da gravidade 10 g =
2
/ m s ; 1 76 atm cmHg = ;
velocidade da luz no vcuo
8
3 10 / c m s = .

QUESTO 01
Ao passar pelo ponto O , um helicptero segue na direo norte
com velocidade v constante. Nesse momento, um avio passa
pelo ponto P , a uma distncia o de O , e voa para o oeste, em
direo aO , com velocidade constante, conforme mostra a figura.
Considerando t o instante em que a distncia d entre o
helicptero e o avio for mnima, assinale a alternativa correta.

Norte
Oeste
u
v
O
P
o

a) A distncia percorrida pelo helicptero no instante em que o
avio alcana o ponto O / u v o .
b) A distncia de helicptero ao ponto O no instante t igual a
2 2
/ v v u o + .
c) A distncia do avio ao ponto O no instante t igual a
2 2 2
/ ( ) v v u o + .
d) O instante t igual a
2 2
/ ( ) v v u o + .
e) A distncia d igual a
2 2
/ u v u o + .
Resoluo Alternativa C
A funo horria do movimento do helicptero, ( ) H t
,
, e do avio,
( ) A t
,
, so dadas por
( ) ( )
H t vt y =
,

( ) ( )
A t ut x = o
,
.
O mdulo da distncia ( ) D t entre eles dada por
( ) ( ) ( ) ( ) ( )
2 2
D t H t A t vt ut = = + o
, ,

( ) ( )
2 2 2 2
2 D t u v t ut = + o + o .
A distncia ser mnima quando ( ) 0
d
D t
dt
= . Assim
( )
2 2
1
2 0
2
d
D t t u v u
dt D
(
= + o =

,
o que resulta em
2 2
u
t
u v
o
=
+
.
Para este instante de tempo,
( )
2 2 2
2 2 2 2
u u v u
A t u
u v u v
o +
= o = o
+ +
( )
2
2 2
v
A t
u v
| |
= o
|
+
\ .


Obsereve que podemos analisar as alternativas do problema
em dois casos especiais, permitindo chegar na alternati va
correta:
a) quando 0 v = ;
b) quando 0 u = .

a)
Quando 0 v = , a menor distncia entre o helicptero e o avio a
prpria distncia 0 d = , que ocorre em / t u = o , pois o helicptero
est parado e o avio voa em direo a ele.
Observe que se colocamos 0 v = na alternativa A, o helicptero
percorre uma distncia infinita:
0
lim
v
u
v

o
= ,
possuindo velocidade nula. Isto um absurdo!
Logo a alternativa A est errada.
Alm disso, se 0 v = , a alternativa D garante que 0 t = , o que est
errado, pois, como vimos, / t u = o .
Assim, a alternativa D est errada.

b)
Quando 0 u = , a menor distncia entre o helicptero e o avio a
prpria distncia inicial d = o , que ocorre em 0 t = , pois o avio
est parado e o helicptero se afasta dele. Neste caso, no instante
de tempo t , a distncia entre o helicptero e o ponto O zero,
pois ele est naquele ponto.
Porm, observe que a alternativa B afirma que quando 0 u = , esta
distncia o , o que no verdade.
Assim, a alternativa B est errada.
Tambm, segundo a alternativa E, 0 d = quando 0 u = , o que est
errado, pois d = o .
Deste modo, a alternati va E est incorreta.

Por eliminao, a alternativa correta a alternativa C.

QUESTO 02
No interior de uma caixa de massa M , apoiada num piso
horizontal, encontra-se fixada uma mola de constante elstica
k presa a um corpo de massa m , em equilbrio na vertical.
Conforme a figura, este corpo tambm se encontra preso a um fio
tracionado, de massa desprezvel, fixado caixa, de modo que
resulte uma deformao b da mola. Considere que a mola e o fio
se encontram no eixo vertical de simetria da caixa. Aps o
rompimento do fio, a caixa vai perder contato com o piso se


k
M
m


a) > + ( ) / b M m g k .
b) > + ( 2 ) / b M m g k .
c) > ( ) / b M m g k .
d) > (2 ) / b M m g k .
e) > ( 2 ) / b M m g k .
Resoluo Alternativa B
Aps o rompimento do fio, o bloco m se deslocar para cima. Para
que a caixa perca o contato com o cho, devemos ter o bloco m
comprimindo a mola e, assim, empurrando a caixa para cima.

Diagrama de Corpo Livre (Caixa):

,
Fel
,
P

>
, ,
| | | | Fel P ; seja x a compresso da mola.

> kx Mg
>
Mg
x
k
(I)

Como no h foras dissipativas, a energia mecnica do sistema se
conserva:

mec final mec inicial
E E =
2 2
1 1
2 2
kx mgh kb + = ; onde
= + h b x =deslocamento vertical total do bloco.


(19) 3251-1012 O ELITE RESOLVE ITA 2013 - FSICA

2

= + +
2 2
1 1
( )
2 2
kb kx mg b x

2 2
1 1
( )
2 2
kb kx mg b x = +

( )
2 2
1
( )
2
k b x mg b x = +
( )
1
2
k b x mg =
2mg
b x
k
= +
Como
Mg
x
k
> , temos:
2mg Mg
b
k k
> + ( ) 2
g
b M m
k
> +

QUESTO 03
Num experimento clssico de Young, d representa a distncia
entre as fendas e D a distncia entre o plano destas fendas e a
tela de projeo das franjas de interferncia, como ilustrado na
figura. Num primeiro experimento, no ar, utiliza-se luz de
comprimento de onda
1
e, num segundo experimento, na gua,
utiliza-se luz cujo comprimento de onda no ar
2
. As franjas de
interferncia dos experimentos so registradas numa mesma tela.
Sendo o ndice de refrao da gua igual a n, assinale a
expresso para a distncia entre as franjas de interferncia
construtiva de ordem m para o primeiro experimento e as de
ordem M para o segundo experimento.

a) ( ) ( )
2 1
/ D M mn nd
b) ( ) ( )
2 1
/ D M m nd
c) ( )
2 1
/ D M mn d
d) ( )
2 1
/ Dn M m d
e) ( )
2 1
/ D Mn m d


Resoluo Alternativa A
No experimento de Young, os mximos de intensidade de ordem k
ocorrem para os ngulos u
k
segundo a condio:

sen sen
k k
k
d k
d

u = u = , com k eZ .

A ordenada y
k
correspondente dada por:

tg
k k
y D = u

Sendo tipicamente ngulos pequenos envolvidos nesse
experimento, podemos aproximar tg sen
k k
u ~ u , de modo que:

k
k
y D
d

=

Para a situao em que o experimento feito na gua, a nica
mudana que devemos usar o comprimento de onda da luz na
gua. Como a frequncia da onda no muda ao refratar do ar para
a gua, temos que:

2
2 2
1 1
GUA AR
AR GUA GUA
AR GUA GUA GUA
v v c c
f f
n n n

= = = =



Assim, a separao Ay entre as franjas de ordem m na primeira
situao (ar) e as franjas de ordem M na segunda situao (gua)
dada por:

1 2 1 GUA
M m
M m M m
y y y D D D D
d d d n d

A = = =


( )
2 1
D
y M n m
d n
A =


QUESTO 04
Num certo experimento, trs cilindros idnticos encontram-se em
contato pleno entre si, apoiados sobre uma mesa e sob a ao de
uma fora horizontal F , constante, aplicada na altura do centro de
massa do cilindro da esquerda, perpendicularmente ao seu eixo,
conforme a figura. Desconsiderando qualquer tipo de atrito, para
que os trs cilindros permaneam em contato entre si, a
acelerao a provocada pela fora deve ser tal que
,
F

a). / (3 3) / 3 g a g s s .
b) 2 / (3 2) 4 / 2 g a g s s .
c) / (2 3) 4 / (3 3) g a g s s .
d) 2 / (3 2) 3 / (4 2) g a g s s .
e) / (2 3) 3 / (4 3) g a g s s .
Resoluo Alternativa A
A acelerao mxima ocorre exatamente quando o cilindro de cima
perde o contato com o cilindro da frente, sendo sustentado apenas
pelo cilindro de trs.
P
,

1 N
,

1x N
,

1y N
,

60




Nessa situao, as foras atuando sobre o cilindro de cima so a fora
peso e a normal devido ao cilindro de trs. As componentes na direo
vertical dessas duas foras devem se equilibrar,
1y
N P =
1
sen60 N mg

=
1
2
3
N mg = , (1)
Pela segunda lei de Newton, a componente horizontal deve igualar
max
ma :
1x R
N F =
1 max
cos 60 N ma

=
1 max
2 N ma = . (2)
Igualando as expresses (1) e (2) obtemos
max
2
2
3
ma mg =
max
3
g
a =

Por outro lado, a acelerao mnima ocorre quando o cilindro da
frente perde o contato com o cilindro de trs.

P
,

2 N
,
2x N
,

N
,

30



Nessa situao temos que a resultante ser apenas a componente
horizontal
2x
N :
2x R
N F =
2 min
sen30 N ma

=
2 min
2 N ma = . (3)

Calculando a resultante sobre o cilindro de cima podemos
encontrar
2
N :

d
D
tela
planodas
fendas


(19) 3251-1012 O ELITE RESOLVE ITA 2013 - FSICA

3

30

30


2 N
,

1x N
,

1 N
,

2x N
,

P
,



1 2 y y
N N P + =
1 2
cos30 cos30 N N mg

+ =

1 2
2
3
N N mg + = (4)

1 2 x x R
N N F =
1 2 min
sen30 sen30 N N ma

=

1 2 min
2 N N ma = (5)
Substituindo (3) em (5), temos:
1 min
4 N ma = . Substituindo este
resultado em (4) e usando novamente (3), vem:
1 2 min
2 2
6
3 3
N N mg ma mg + = =
min
3 3
g
a =
Assim a resposta
min max
a a a s s

3 3 3
g g
a s s

QUESTO 05
Duas partculas, de massas m e M , esto respectivamente
fixadas nas extremidades de uma barra de comprimento L e
massa desprezvel. Tal sistema ento apoiado no interior de
uma casca hemisfrica de raio r , de modo a se ter equilbrio
esttico com m posicionado na borda P da casca e M , num
ponto Q , conforme mostra a figura. Desconsiderando foras de
atrito, a razo / m M entre as massas igual a

Q
L
M
r O
P
m


a)
2 2 2
( 2 ) / (2 ) L r r .
b)
2 2 2
(2 3 ) / (2 ) L r r .
c)
2 2 2 2
( 2 )( ) L r r L .
d)
2 2 2 2
(2 3 ) / ( ) L r r L .
e)
2 2 2 2
(3 2 ) / ( 2 ) L r L r .
Resoluo Alternativa A

Diagramas de Corpos Livres para m e M :

M
O
( ) Mg
u
2u
( ) mg
1
,
P
2
,
P
( ) F
( ) F
1
,
N
Q
P
m
2
,
N
u
u


y
x

Equilbrio Esttico do corpo m :

Dir. x :
1
cos N F = u (I)
Dir. y : sen mg F = u (II)

Equilbrio Esttico do corpo M :
Dir. x :
2
cos2 cos N F u = u (III)
Dir. y :
2
sen2 sen N F Mg u = u + (IV)

Fazendo
( )
( )
II
I
, temos:

1
tg
mg
N
= u =>
1
tg
mg
N =
u
(V)

Fazendo
( )
( )
IV
III
, temos:

( )
sen
tg 2
cos cos
u
u = +
u u
F Mg
F F
(VI)
Comparando, (I) e (V) temos:

cos
tg
u =
u
mg
F (VII)
Substituindo (VII) em (VI)
( ) tg 2 tg
tg
u = u +
| |
|
u
\ .
Mg
mg

( ) tg 2 tg tg u = u + u
M
m

( ) tg 2 tg (1 ) u = u +
M
m

2
2tg
tg (1 )
(1 tg )
M
m
u
= u +
u


Dividindo ambos os membros por tgu (tg 0) u = , temos:

2
2
1
1 tg
M
m
= +
u

2
2
1
1 tg
M
m
= =
u

2
2
2 1 tg
1 tg
M
m
+ u
=
u

2
2
1 tg
1 tg
M
m
+ u
=
u

2
2 2 2
2
2 2 2 2
2
sen
1
1 tg cos sen
cos
sen 1 tg cos sen
1
cos
m
M
u

u u u
u
= = =
u + u u + u
+
u

2 2 2
cos sen 2cos 1
m
M
= u u = u

Da geometria do problema, temos:


u
P
r
O
r
Q
2
L
2
L


2
cos
2
L
L
r r
| |
|
\ .
u = =


Portanto:

2
2
2
2
2 1 2 1
2 4
m L L
M r r
| |
= =
|
\ .

2 2
2
( 2 )
2
m L r
M r

=



(19) 3251-1012 O ELITE RESOLVE ITA 2013 - FSICA

4

QUESTO 06
Uma corda, de massa desprezvel, tem fixada em cada uma de
suas extremidades, F eG , uma partcula de massa m . Esse
sistema encontra-se em equilbrio apoiado numa superfcie
cilndrica sem atrito, de raior , abrangendo um ngulo de 90 e
simetricamente disposto em relao ao pice P do cilindro,
conforme mostra a figura. Se a corda for levemente deslocada e
comea a escorregar no sentido anti-horrio, o ngulo FP u
em que a partcula na extremidade F perde contato com a
superfcie tal que

P
m
G
45

45

O
r
m
F


a) 2cos 1 u = .
b) 2cos sen 2 u u = .
c) 2sen cos 2 u + u = .
d) 2cos sen 2 u + u = .
e) 2cos sen 2 / 2 u + u = .
Resoluo Alternativa D
Condio Inicial (em repouso):

P
m
G
45

45

O
r
m
F
2
cos45
2
h r r = =


. .(1) . .(2)
( )
MEC POT G POT G
E sistema E E = +
2 2
( ) 2
2 2
MEC
E sistema mg r mg r mgr = + = eq. (I)
(Energia Potencial Gravitacional relativa ao centro da superfcie
cilndrica)
Condio final (perda de contato de (1);
1
0 N =
, ,
)

m
u
O
r
m
sen r u
u
cos r u
T
,
P
,
r
u


Os mdulos das velocidades das partculas so iguais, pois esto
ligadas por um fio. Seja v o mdulo dessas velocidades:

. .(1) . .(2) (1) (2)
' ( )
MEC POT G POT G CIN CIN
E sistema E E E E = + + +
2 2
1 1
' ( ) cos sen
2 2
MEC
E sistema m g r m g r mv mv = u + u + +
2
' ( ) cos sen
MEC
E sistema m g r m g r m v = u + u + (II)

A componente centrpeta da resultante dada por:

2
cos
cp
m v
R P
r

= = u
2
cos v r g = u (III)

Como nesse sistema no h foras dissipativas, de (I) e (II), pela
conservao da Energia Mecnica, temos:
'
MEC MEC
E E =
2
cos sen 2 m g r m g r m v m g r u + u + = ;
Da equao (III):
2
cos v r g = u
cos sen ( cos ) 2 m g r m g r m r g m g r u + u + u =
Dividindo por (m g r), temos:
cos sen cos 2 u + u + u =

Logo, 2cos sen 2 u + u = .

QUESTO 07
Uma pequena bola de massa m lanada de um pontoP contra
uma parede vertical lisa com uma certa velocidade
0
v , numa
direo de ngulo o em relao horizontal. Considere que aps
a coliso a bola retorna ao seu ponto de lanamento, a uma
distncia d da parede, como mostra a figura. Nestas condies, o
coeficiente de restituio deve ser


P
o
m
0
v


a) = o
2
0
/ ( 2 ) e gd v sen gd .
b) = o
2
0
2 / ( cos2 2 ) e gd v gd .
c) = o
2
0
3 / (2 2 2 ) e gd v sen gd .
d) = o
2
0
4 / ( cos2 ) e gd v gd .
e) = o
2
0
2 / ( tan2 ) e gd v gd .
Resoluo Alternativa A
O coeficiente de restituio dado por:

vx depois
vx antes
antes v
,

depois v
,


2 1
2
1
x depois
x antes
d
V t t
e
V t d
t
| |
|
A A
\ .
= = =
A | |
|
A
\ .
(I)

0 1
1 0
cos
cos
xantes
d d
v v t
t v
= o = A =
A o
(II)

Note que
1 2 voo
t t t A = A + A (tempo total de voo).

Como a parede vertical e lisa, a velocidade na direo vertical no
afetada pela coliso. Logo, o
voo
t A o mesmo de um lanamento
oblquo sem a parede:
y oy
v v gt =
Aplicando-se a equao acima para o ponto de altura mxima da
trajetria:
0 sen .
2
voo
o
t
v g
A | |
= o
|
\ .

0
2 sen
voo
v
t
g
o
A = (III)
2 1 voo
t t t A = A A
2
0 0
2
0 0
2 sen 2 sen cos
cos cos
v v gd d
t
g v v g
o o o
A = =
o o



(19) 3251-1012 O ELITE RESOLVE ITA 2013 - FSICA

5

2
0
2
0
sen2
cos
v gd
t
v g
o
A =
o
(IV)

Substituindo (IV) e (II) em (I), obtemos:

1
2 2
2 0 0
0
cos
( sen2 ) sen2
cos
o
d
v t gd
e e
t v gd v gd
v g
| |
|
o A
\ .
= = =
A o | | o
|
o
\ .


QUESTO 08
A figura mostra um sistema, livre de qualquer fora externa, com
um mbolo que pode ser deslocado sem atrito em seu interior.
Fixando o mbolo e preenchendo o recipiente de volume V com
um gs ideal a presso P, e em seguida liberando o mbolo, o gs
expande-se adiabaticamente. Considerando as respectivas
massas m
c
, do cilindro, e m
e
, do mbolo, muito maiores que a
massa m
g
do gs, e sendo o expoente de Poisson, a variao da
energia interna AU do gs quando a velocidade do cilindro for v
c

dada aproximadamente por

a) 3 / 2 PV

.
b) 3 / (2( 1)) PV .
c)
2
( ) / (2 )
c e c c e
m m m v m + .
d)
2
( ) / 2
c e c
m m v + .
e)
2
( ) / (2 )
e e c c c
m m m v m +

Resoluo Alternativa C
Sendo uma expanso adiabtica (calor absorvido pelo gs 0 Q = ),
segue da Primeira Lei da Termodinmica que:

0
Q W U
U W
Q
= + A
A =

,

onde W o trabalho realizado pelo gs, transferido integralmente na
forma de energia cintica ao sistema cilindro + mbolo.





O enunciado nos diz que o sistema estudado encontra-se livre de
quaisquer foras externas. Dessa forma, podemos afirmar que a
quantidade de movimento P do sistema se conserva entre a primeira
situao (repouso) e a segunda situao (instante em que o cilindro
est com velocidade v
c
):

0
c c
INICIAL FINAL c c e e e
e
m v
P P m v m v v
m

= = =

Note que, devido massa m
g
do gs ser muito menor que as outras
massas consideradas, ela no precisa ser levada em conta para os
efeitos estudados.
A partir disso, o clculo do trabalho W feito utilizando o teorema do
trabalho-energia cintica:

2 2
0
2 2
c c e e
c
m v m v
W E
| |
= A = +
|
\ .

2 2 2 2
2
1
2 2 2
c c e c c c c c
e e
m v m m v m v m
W W
m m
| |
= + = +
|
\ .


Ento:

2
1
2
c c c
e
m v m
U W
m
| |
A = = +
|
\ .
( )
2
2
c
c e c
e
v
U m m m
m
A = +






QUESTO 09
Uma rampa macia de 120 kg inicialmente em repouso, apoiada
sobre um piso horizontal, tem sua declividade dada por
u = tan 3/ 4 . Um corpo de 80 kg desliza nessa rampa a partir do
repouso, nela percorrendo 15 m at alcanar o piso. No final
desse percurso, e desconsiderando qualquer tipo de atrito, a
velocidade da rampa em relao ao piso de aproximadamente

a) 1 m/s.
b) 3 m/s.
c) 5 m/s.
d) 2 m/s.
e) 4 m/s.
Resoluo Alternativa C
Para resolver este problema empregaremos as seguintes
quantidades:
U Ux =
,
: velocidade horizontal da rampa em relao a um
observador parado no solo;

x y
v v x v y = +
,
: velocidade do corpo em relao rampa;
( )

x y
V v U v U x v y = = +
, ,
,
: velocidade do corpo em relao a um
observador parado no solo.


V
,

U
,



Como o corpo percorre um trajeto de 15 m ao longo da rampa,
temos que a altura e o comprimento desta rampa so,
respectivamente, 9 m e 12 m, uma vez que o ngulo da rampa
dado por
3
tan
4
u = :

15 m
9 m
12 m
u

Para a descida, duas quantidades iro se conservar:
i) a energia mecnica do sistema, uma vez que os
atritos so desprezveis;
ii) a quantidade de movimento na direo horizontal,
uma vez que no h foras externas nesta direo.
Alm disso, temos a relao
iii)
3
tan
4
y
x
v
v
u = = . (1)

Designaremos por 80 kg m = a massa do corpo, e por 120 kg M =
a massa da rampa. Temos para a conservao da quantidade de
movimento na direo horizontal, no referencial do solo:
( )
x
m v U MU =
( )
x
U
v M m
m
= + (2)
v
c
v
e
V
m
g
m
c
m
e


(19) 3251-1012 O ELITE RESOLVE ITA 2013 - FSICA

6

Como a altura inicial do corpo antes da queda 9 m h = , a
conservao de energia nos garante:
( )
2 2
2
2 2
2 2 2 2
y
x
M m M m
mgh U V U v U v
(
= + = + +

, ,
.
Substituindo (1) e (2) na equao acima:
( ) ( )
2 2
2
9
2 2 16
M m U U
mgh U M m U M m
m m

( (
= + + + +
`
( (


)

( )
2
2 2 2
1 9
2 2 16
M
mgh U U M M m
m
(
= + + +
(


( )
2
2
2
1 9
2 2 16
mgh
U
M
M M m
m
=
(
+ + +
(


2
32
1 25 9
gh
U
M M
m m
=
| | (
+ +
| (
\ .

Agora, substituindo os valores, e notando que a razo
M
m
dada
por:
120 3
80 2
M
m
= = , temos
2
32 10 9 11520
24,77
3 3 465
1 25 9
2 2
U

= = ~
| | (
+ +
| (
\ .

5 m/s U ~ .

QUESTO 10
Certo produto industrial constitui-se de uma embalagem rgida
cheia de leo, de dimenses x x L L d , sendo transportado numa
esteira que passa por um sensor capacitivo de duas placas
paralelas e quadradas de lado L , afastadas entre si de uma
distncia ligeiramente maior que d , conforme a figura. Quando o
produto estiver inteiramente inserido entre as placas, o sensor
deve acusar um valor de capacitncia
0
C . Considere, contudo,
tenha havido ante um indesejado vazamento de leo, tal que a
efetiva medida da capacitncia seja
0
3/4 C C = . Sendo dadas as
respectivas constantes dieltricas do leo, 2 k = ; e do ar, 1 k =
ar
,
e desprezando o efeito da constante dieltrica da embalagem,
assinale a percentagem do volume de leo vazado em relao ao
seu volume original.

sensor capacitivo
L
L
> d
~
L
L
d
esteira
produto

a) 5%.
b) 50%.
c) 100%.
d) 10%.
e) 75%.
Resoluo Alternativa B
Quando a embalagem que sofreu derramamento passa por entre
as placas do sensor, teremos na verdade uma associao em
paralelo de capacitores.
Observe os esquemas abaixo. O esquema esquerda representa
o sensor capacitivo e o leo; o esquema da direita uma vista
lateral do mesmo sensor.


L
L
d
x


x
L
d


Perceba que a figura direita equivalente a uma associao em
paralelo de capacitores (conforme esquematizado abaixo) cuja
capacitncia equivalente
eq 1 2
C C C = + .

0
k c
ar 0
k c
1
C
2
C

Como a capacitncia de um capacitor plano dada por
0
k A
C
d
c
= , podemos calcular a capacitncia equivalente deste
conjunto de capacitores da seguinte forma:

( )
0 1 ar 0 2 0 0
2 1 c c c + c
= + =
eq
k A k A L x L L x
C
d d d


( )
0
2
eq
L
C x L x
d
c
= +
( )
0
eq
L
C x L
d
c
=

Como foi dito que
0
3/4 = =
eq
C C C sendo

2
0 0
0 0
2 k A L
C C
d d
c c
= =

Assim:

( )
2
0 0 0
3 2 3 3
4 4 2
eq
C L L
C x L x L L
d d
c c
= = =

2
L
x =

Dessa forma, como o leo preenche metade do recipiente, o
volume ocupado a metade:
2
total
leo
V L d L
V L x d x

= =

leo total
0,5 V V =

QUESTO 11
O circuito mostrado na figura constitudo por um gerador com
f.e.m. c e um resistor de resistncia R . Considere as seguintes
afirmaes, sendo a chave S fechada:

S
c
R


I Logo aps a chave S ser fechada haver uma f.e.m.
autoinduzida no circuito.
II Aps um tempo suficientemente grande cessar o fenmeno
de autoinduo no circuito.
III A autoinduo no circuito ocorrer sempre que houver
variao da corrente eltrica no tempo.
Assinale a alternativa verdadeira.
a) Apenas a I correta.
b) Apenas a II correta.
c) Apenas a III correta.
d) Apenas a II e a III so corretas.
e) Todas so corretas.


(19) 3251-1012 O ELITE RESOLVE ITA 2013 - FSICA

7

Resoluo Alternativa E
O circuito representado na figura do enunciado delimita uma rea
fechada que denominaremos S . Ao passar uma corrente no
circuito, digamos que ela circule no sentido anti-horrio, um campo
magntico perpendicular pgina e saindo dela passa a
atravessar esta rea S , gerando um fluxo magntico
B
u .
O conceito por trs da autoinduo que variaes deste fluxo
magntico induzem uma fora eletromotriz no circuito, ou seja, ao
mudarmos a corrente que o atravessa, esta variao produz um
potencial.
Analisemos as afirmativas:
I. Verdadeira
Quando fechamos a chave, variamos a corrente, e como explicado
anteriormente, haver f.e.m. induzida no circuito.
II. Verdadeira
Quando fechamos a chave, a corrente varia rapidamente e com o
passar do tempo seu valor tende a se estabilizar em / i R = c .
Quando a corrente constante, o fluxo magntico
B
u tambm
constante, e no h f.e.m. induzida.
III. Verdadeira
Sempre que variamos a corrente, o fluxo
B
u tambm varia, e
produz f.e.m., ocorrendo autoinduo.

Devido ao exposto acima, a alternativa correta a E, pois todas as
afirmativas esto corretas.

Nota: Se consideramos um circuito rgido, somente variaes na
corrente podem variar o fluxo magntico que atravessa o circuito.
Se permitirmos variaes na forma do circuito, por exemplo,
mudando sua rea, tambm possvel produzir uma f.e.m.
induzida sem variar a corrente. Sob esta hiptese de no rigidez
do circuito, as afirmativas II e III estariam incorretas.

QUESTO 12
Um raio horizontal de luz monocromtica atinge um espelho plano
vertical aps incidir num prisma com abertura de 4 e ndice de
refrao 1,5 n = . Considere o sistema imerso no ar e que tanto o
raio emergente do prisma como o refletido pelo espelho estejam
no plano do papel, perpendicular ao plano do espelho, como
mostrado na figura. Assinale a alternativa que indica
respectivamente o ngulo e o sentido em que deve ser girado o
espelho em torno do eixo perpendicular ao plano do papel que
passa pelo ponto O , de modo que o raio refletido retorne
paralelamente ao raio incidente no prisma.


Espelho
O
4


a) 4, sentido horrio.
b) 2, sentido horrio.
c) 2, sentido antihorrio.
d) 1, sentido horrio.
e) 1, sentido antihorrio.
Resoluo Alternativa D
O enunciado desta questo falho ao:
- no deixar claro se o raio horizontal antes ou aps incidir
no prisma. Poderamos ainda supor que a vista do enunciado
da questo superior, de modo que o plano do papel seria
horizontal, assim todos os raios seriam horizontais,
entretanto, um dado importante para a resoluo da questo
o paralelismo entre o raio incidente no prisma e o eixo
perpendicular ao espelho, o que no dado na questo;
- no fornecer informaes suficientes para afirmar que o
ngulo de incidncia no prisma pequeno. O esquema
mostra um prisma com bissetriz do ngulo de abertura
aparentemente perpendicular ao ngulo de incidncia,
porm, claramente a figura no est em escala, como se
pode notar ao observar o ngulo de 4, cujo desenho muito
maior que a medida indicada.

Apesar dos comentrios acima, resolveremos a questo assumindo
que o ngulo de incidncia no prisma pequeno e que, alm de a
vista ser lateral, o raio horizontal antes de incidir no prisma. Assim, o
raio incidente i no prisma e a reta N
E
, normal ao espelho, so
paralelos.

Na figura abaixo, observe que o desvio A do raio de luz ao atravessar
o prisma igual ao ngulo
1
u (alternos internos), que igual ao
ngulo
2
u (segunda lei da reflexo).

r1
r2 i2
i1
A
N1
N2
O
A
1
u
2
u
NE
i

Ao aplicar a lei de Snell, assumindo que os ngulos so muito
pequenos, vale a aproximao senu u ~ :
1 1
1 sen sen i n r = , logo:
1 1
i n r = (1)

Analogamente:
2 2
i n r = (2)

E ainda:
1 2
r r A + = (3)

De (1), (2) e (3):
1 2 1 2 1 2
( ) i i n r r i i n A + = + + = (4)

Sabemos que o desvio dado por:
1 1 2 2 1 2
i r i r i i A A = + = + (5)

Substituindo (4) em (5) e lembrando que n = 1,5 e A = 4, temos:
1 2
( 1) i i A n A A A n A = + = =
4 0,5 A =
2 A =

Agora, para que o raio emergente do espelho volte paralelo ao raio
incidente no prisma, necessrio que o raio gire
2
u e, portanto, o
espelho gire
2
2
u
, como pode ser visto na figura abaixo:



(19) 3251-1012 O ELITE RESOLVE ITA 2013 - FSICA

8

O
2
u
NE
2
u
2
2
u

NE

Como
2
2 u = A = , precisamos girar o espelho de 1 no sentido
horrio.

QUESTO 13
Um prato plstico com ndice de refrao 1,5 colocado no interior
de um forno de micro-ondas que opera a uma frequncia de
2,5 10
9
Hz. Supondo que as micro-ondas incidam
perpendicularmente ao prato, pode-se afirmar que a mnima
espessura deste em que ocorre o mximo de reflexo das micro-
ondas de
a) 1,0 cm. b) 2,0 cm. c) 3,0 cm.
d) 4,0 cm. e) 5,0 cm.
Resoluo Alternativa B
Como as micro-ondas so ondas eletromagnticas, seu
comprimento de onda no plstico pode ser calculado atravs da
definio do ndice de refrao absoluto:

P P
P P P P P
c c c
n
v f n f
= = =



Como a frequncia da onda no muda ao passar de um meio para
outro, segue que:

8
9
3 10
0,08 m 8 cm
1,5 2,5 10
P
P P
c
n f

= = = =



Considere agora a representao a seguir de dois raios de onda
que incidem sobre o prato plstico. Embora a incidncia seja
perpendicular superfcie, fizemos o desenho considerando os
raios oblquos para maior clareza.


Um dos raios representados (raio 1) aquele que reflete
diretamente sobre a superfcie superior do plstico, e nesse caso a
reflexo se d COM inverso de fase, dado que o plstico um
meio mais refringente que o ar.

O outro raio representado (raio 2) aquele que refrata para dentro
do plstico, sofre reflexo SEM inverso de fase na superfcie
inferior do plstico (pois o ar menos refringente que o plstico) e
refrata de volta para o ar na superfcie superior.

Para que a reflexo das micro-ondas seja favorecida, esses dois
raios que emergem da superfcie superior do plstico devem sofrer
interferncia construtiva. Considerando que j houve uma
defasagem de meio comprimento de onda entre esses raios,
devida inverso da fase na reflexo do raio 1, a menor
espessura para a qual eles interferem construtivamente aquela
que acrescenta mais meio comprimento de onda de defasagem
em razo da diferena de caminhos entre os raios. Temos que:

8
2
2 4 4
P P
e e

= = = 2 cm e =
QUESTO 14
Considere o circuito eltrico mostrado na figura formado por quatro
resistores de mesma resistncia, 10 R = O, e dois geradores
ideais cujas respectivas foras eletromotrizes so
1
30 c = V e
2
10 c = V. Pode-se afirmar que as correntes i
1
, i
2
, i
3
e i
4
nos trechos
indicados na figura, em ampres, so respectivamente de
1
c
2
c
R R
R
R
i
4
i
1
i
3
i
2

a) 2, 2/3, 5/3 e 4.
b) 7/3, 2/3, 5/3 e 4.
c) 4, 4/3, 2/3 e 2.
d) 2, 4/3, 7/3 e 5/3.
e) 2, 2/3, 4/3 e 4.
Resoluo Alternativa B
Podemos aplicar as Leis de Kirchhoff para resolver este problema:
Pela primeira Lei de Kirchhoff, obtemos que:
1 2 3
i i i = + eq. (1)
Aplicando a segunda Lei de Kirchhoff nas malhas que compem o
circuito dados, obteremos mais trs equaes.
Para a malha superior, obtemos:


1
c
R
R
i
1
i
2
I
1 1 2 1 2
0 30 10 10 0 +c = + = R i R i i i
1 2
3 + = i i eq. (2)

Para a malha inferior esquerda, temos:

1
c
2
c
R
i
4
II

1 2 4 4
0 30 10 10 0 c c + = + = R i i
4
4 = i eq. (3)

Para a malha inferior direita, encontramos:

2
c
R
R
i
3
i
2
III

2 2 3 2 3
0 10 10 10 0 +c + = + + = R i R i i i
3 2
1 = i i eq. (4)

As quatro equaes acima nos fornecem um sistema de quatro
variveis:
e
Plstico
Raio
2
Raio
1


(19) 3251-1012 O ELITE RESOLVE ITA 2013 - FSICA

9

1 2 3
1 2
4
3 2
3
4
1
= +

+ =

i i i
i i
i
i i


Resolvendo o sistema obtemos:

1
7/3 A i = ,
2
2/3 A i = ,
3
5/3 A i = e
4
4 A i = .

QUESTO 15
A figura mostra duas cascas esfricas condutoras concntricas no
vcuo, descarregadas, em que a e c so, respectivamente, seus
raios internos, e b e d seus respectivos raios externos. A seguir,
uma carga pontual negativa fixada no centro das cascas.
Estabelecido o equilbrio eletrosttico, a respeito do potencial nas
superfcies externas das cascas e do sinal da carga na superfcie
de raio d, podemos afirmar, respectivamente, que


d
c
b
a


a) > ( ) ( ) V b V d e a carga positiva.
b) < ( ) ( ) V b V d e a carga positiva.
c) = ( ) ( ) V b V d e a carga negativa.
d) > ( ) ( ) V b V d e a carga negativa.
e) < ( ) ( ) V b V d e a carga negativa.
Resoluo Alternativa E
Ao posicionar uma carga negativa Q no centro das esferas, temos as
seguintes cargas induzidas:

A carga induzida na interface de raio a +Q.
A carga induzida na interface de raio b Q.
A carga induzida na interface de raio c +Q.
A carga induzida na interface de raio d Q.

d
c
b
a
-Q
-Q
-Q
+Q
+Q


A primeira concluso que a carga na superfcie de raio d negativa.

Prximo carga central negativa, o potencial negativo e seu mdulo
muito grande.
No interior (regio oca) da casca esfrica menor, o campo eltrico de
aproximao, apontando para o centro, o que indica que cada ponto
que est a uma distncia D a s tem seu potencial aumentando,
conforme se afasta do centro.
Na parte metlica a D b s s o potencial constante.
Na prxima regio ( b D c s s ), o potencial volta a aumentar, pois a
carga interna negativa e o campo eltrico volta a ser de aproximao
para o centro.
Na ltima parte metlica, o potencial volta a ser constante com o
aumento da distncia.
Fora da esfera, a carga interna negativa e o potencial volta a
aumentar, tendendo a zero para D muito grande.

Assim, temos qualitativamente:

V
D
a b c d
V(d)
V(b)

Conclui-se, ento, que ( ) ( ) V b V d <

Soluo algbrica alternativa para o clculo dos potenciais:

Cada conjunto de cargas (totalizando q) em formato de casca esfrica
de raio R provoca um potencial em um ponto que est a uma distncia
D do centro igual a:

i
K q
V
R

= se o ponto interior casca


esfrica ou est sobre ela ( D R s ):

D
q
R

e
K q
V
D

= se o ponto exterior casca


esfrica ( D R > ):

D
q
R

Assim, podemos calcular os potenciais V(b) e V(d):

( ) ( ) ( ) ( ) ( )
( )
Q K K K K K
V
Q Q Q
b
Q
b b b c d
+
= + + +
+
+
( ) ( ) ( )
( )
K K K
V b
b
Q Q
c
Q
d

+
+
+

=

( ) ( ) ( ) ( ) ( )
( )
Q K K K K K
V
Q Q Q
d
Q
d d d d d
+
= + + +
+
+
( )
( )
K
V d
d
Q
=

Observe que a diferena entre V(b) e V(d) :
( ) ( ) ( ) ( )
( ) ( )
K K K Q Q K
V b V d
Q Q
b c d d
| | | |
= + +
| |
\

. \
+
.


1 1
( ) ( ) Q V b V d K
c b
| |
=
|
\ .


Como c b > , ento
1 1 1 1
0
c b c b
| |
< <
|
\ .
. Da:
( ) ( ) 0 ( ) ( ) V b V d V b V d < <


(19) 3251-1012 O ELITE RESOLVE ITA 2013 - FSICA

10

QUESTO 16
Um recipiente contm dois lquidos homogneos e imiscveis, A e
B, com densidades respectivas
A
e
B
. Uma esfera slida, macia
e homognea, de massa m = 5 kg, permanece em equilbrio sob
ao de uma mola de constante elstica k = 800 N/m, com metade
de seu volume imerso em cada um dos lquidos, respectivamente,
conforme a figura. Sendo
A
= 4 e
B
= 6, em que a
densidade da esfera, pode-se afirmar que a deformao da mola
de

a) 0 m.
b) 9/16 m.
c) 3/8 m.
d) 1/4 m.
e) 1/8 m.

Resoluo Alternativa D
As foras que atuam sobre a esfera slida so:

- P
,
: o peso da esfera, vertical para baixo;
-
A
E
,
: o empuxo exercido pelo lquido A, vertical para cima;
-
B
E
,
: o empuxo exercido pelo lquido B, vertical para cima;
-
EL
F
,
: a fora exercida pela mola.

Num primeiro momento, no podemos afirmar se a fora elstica
tem sentido vertical para cima ou para baixo. Para decidir,
necessrio comparar o mdulo da fora peso com o mdulo do
empuxo aplicado sobre a esfera pelos lquidos.

A fora peso tem intensidade dada por:

5 10 50 N P m g = = =
, ,


Sejam V o volume da esfera, e V
A
e V
B
os volumes deslocados
dos lquidos A e B, respectivamente. De acordo com o enunciado,
temos que:

2
A B
V
V V = =

Assim, os empuxos aplicados por cada lquido sobre a esfera tm
intensidades dadas por:

( )
( )
4 2 2 2 5 10 100 N
2
6 3 3 3 5 10 150 N
2
A A A
B B B
V
E g V g V g m g
V
E g V g V g m g

= = = = = =

= = = = = =

, , , , ,
, , , , ,

O empuxo total sobre a esfera tem intensidade igual a:

100 150 250 N
A B
E E E = + = + =
, , ,


Como E P >
, ,
, para manter o sistema em equilbrio, a fora
elstica deve ter sentido para baixo, e agora podemos montar o
diagrama de foras agindo sobre a esfera:


Assim, a mola deve estar distendida de um comprimento x tal que:

800 250 50 + = = =
, , , , ,
EL
F P E k x E P x
1
m
4
x =


QUESTO 17
Diferentemente da dinmica newtoniana, que no distingue
passado e futuro, a direo temporal tem papel marcante no nosso
dia-a-dia. Assim, por exemplo, ao aquecer uma parte de um corpo
macroscpico e o isolarmos termicamente, a temperatura deste se
torna gradualmente uniforme, jamais se observando o contrrio, o
que indica a direcionalidade do tempo. Diz-se ento que os
processos macroscpicos so irreversveis, evoluem do passado
para o futuro e exibem o que o famoso cosmlogo Sir Arthur
Eddington denominou de seta do tempo. A lei fsica que melhor
traduz o tema do texto

a) a segunda lei de Newton.
b) a lei de conservao da energia.
c) a segunda lei da termodinmica.
d) a lei zero da termodinmica.
e) a lei de conservao da quantidade de movimento.
Resoluo Alternativa C
a) Incorreta. A segunda lei de Newton relaciona uma fora resultante
aplicada a um corpo com sua mudana de estado de movimento
(quantidade de movimento) ao longo do tempo, tendo relao de
direcionalidade com o tempo, pois, se soubermos a direo e o
sentido da fora, podemos observar dois estados de movimento e
dizer qual dos dois ocorreu antes e qual ocorreu depois. Esse
processo, no entanto, no o que melhor traduz o tema do texto, pois
no est relacionado a temperaturas e isolamento trmico.
b) Incorreta. A conservao de energia diz que a energia de um
sistema isolado antes de um processo fsico igual energia aps o
processo. Sendo os dois estados idnticos (mesma energia), no
podemos distinguir qual ocorreu antes e qual ocorreu depois. No h,
portanto, relao de direcionalidade com o tempo.
c) Correta. A segunda lei da termodinmica se refere justamente ao
aumento de entropia de um sistema isolado. No caso do corpo
aquecido apenas em uma parte e isolado, seu estado menos
entrpico do que aquele cuja temperatura uniforme, ento a referida
lei diz que nunca ser observado, com o passar do tempo, um
acmulo de energia trmica em apenas um ponto do corpo. Dessa
forma, distribuir uniformemente a temperatura de um sistema isolado
um processo irreversvel, pois ocorre espontaneamente, ao contrrio
do acmulo de energia, que precisa ser um processo forado (requer
uma fonte externa de energia para ocorrer).
d) Incorreta. A lei zero da termodinmica diz que se um corpo A est
em equilbrio trmico (tem mesma temperatura) que um corpo B e o
corpo B est em equilbrio trmico com um corpo C, ento A e C esto
em equilbrio trmico entre si. Essa lei no tem nenhuma relao com
a passagem do tempo, pois est aplicada a estados (instantneos).
e) Incorreta. A lei de conservao da quantidade de movimento,
analogamente da conservao de energia, no possui relao de
direcionalidade com o tempo.

QUESTO 18
Num experimento que usa o efeito fotoeltrico ilumina-se a
superfcie de um metal com luz proveniente de um gs hidrognio
cujos tomos sofrem transies do estado n para o estado
fundamental. Sabe-se que a funo trabalho| do metal igual
metade da energia de ionizao do tomo de hidrognio cuja
energia do estado n dada por
2
1 n
E E n = . Considere as
seguintes afirmaes:

I - A energia cintica mxima do eltron emitido pelo metal
2
1 1
/ / 2
c
E E n E = .
II - A funo trabalho do metal
1
/ 2 E | = .
III - A energia cintica mxima dos eltrons emitidos aumenta com
o aumento da frequncia da luz incidente no metal a partir da
frequncia mnima de emisso.

Assinale a alternativa verdadeira.
a) Apenas a I e III so corretas.
b) Apenas a II e III so corretas.
c) Apenas a I e II so corretas.
d) Apenas a III correta.
e) Todas so corretas.


m
A
B
k
P
,

EL
F
,

E
,



(19) 3251-1012 O ELITE RESOLVE ITA 2013 - FSICA

11

Resoluo Alternativa E
Para resolver este exerccio devemos nos lembrar que a energia
f
E
do fton emitido pelo tomo de hidrognio quando vai de um estado
excitado n para o estado fundamental :

1 f n
E E E =
1
1 2 f
E
E E
n
=

Para que o tomo seja ionizado devemos lembrar que necessria
uma energia
i
E suficiente para retirar o seu eltron do estado
fundamental e lev-lo para um estado onde n :

1
1 1 2
lim 0
i
n
E
E E E
n

| |
= =
|
\ .
1 i
E E =

O enunciado nos diz que a funo trabalho | do metal igual
metade da energia de ionizao. Dessa forma:

2
i
E
| =
1
2
E
| =

A energia cintica mxima
c
E de um eltron ejetado no efeito
fotoeltrico dada pela energia do fton incidente menos a energia
necessria para vencer a funo trabalho | do metal:

c f
E E = |
1 1
1 2
2
c
E E
E E
n
| |
=
|
\ .

1 1
2
2
c
E E
E
n
=

Podemos notar que, como a funo trabalho do metal constante,
qualquer acrscimo na energia do fton incidente provoca acrscimo
na energia cintica mxima do eltron emitido.
A energia de um fton dada na forma:

f
E h f =

Onde h a constante de Planck e f a frequncia do fton.
Percebemos ento que a energia do fton cresce quando cresce sua
frequncia. Se
c
E cresce com
f
E e
f
E cresce com f , ento
c
E
cresce com f . Isso torna as trs afirmaes verdadeiras.

QUESTO 19
Uma espira circular de raio R percorrida por uma corrente
eltrica i criando um campo magntico. Em seguida, no mesmo
plano da espira, mas em lados opostos, a uma distncia 2R do
seu centro colocam-se dois fios condutores retilneos, muito
longos e paralelos entre si, percorridos por correntes i
1
e i
2
no
nulas, de sentidos opostos, como indicado na figura. O valor de i e
o seu sentido para que o mdulo do campo de induo resultante
no centro da espira no se altere so respectivamente


a) ( )( )
1 2
1/ 2 i i i = t + e horrio.
b) ( )( )
1 2
1/ 2 i i i = t + e antihorrio.
c) ( )( )
1 2
1/ 4 i i i = t + e horrio.
d) ( )( )
1 2
1/ 4 i i i = t + e antihorrio.
e) ( )( )
1 2
1/ i i i = t + e horrio.
Resoluo Alternativa D
Inicialmente, temos um vetor induo magntica B
,
no centro da
espira de intensidade dada por:

2
i
B
R

,


Quando os fios retilneos passam a ser percorridos pelas correntes
i
1
e i
2
, elas geram no plano da figura vetores induo magntica
1
B
,
e
2
B
,
perpendiculares a esse plano, com os sentidos indicados
a seguir:


Assim, no centro da espira circular, ambas as correntes geram
vetores induo magntica entrando no plano da figura, de
intensidade dada por:

( )
1 2
1 2
1 2
2 2 2 2 4
i i
i i
B B
R R R
+

+ = + =
t t t
, ,


Observe que, se o vetor induo magntica criado pela corrente i
que circula na espira tivesse o mesmo sentido desses outros dois
campos (ou seja, entrando no plano da figura), no seria possvel
manter o mdulo do vetor induo magntica resultante igual ao
da situao anterior, pois seria o da situao anterior somado com
os mdulos desses dois novos vetores.

Portanto, a primeira concluso que o vetor induo magntica
criado pela corrente i no centro da espira deve estar saindo do
plano da figura, e para tanto a corrente na espira deve estar
circulando no sentido anti-horrio.


Alm disso, a nica maneira de manter constante o mdulo do
vetor induo magntica resultante no centro da espira entre as
duas situaes se forem vetores de sentidos opostos, ou seja,
saindo do plano da figura na primeira situao (quando s temos o
vetor induo magntica criado pela corrente i) e entrando no
plano da figura na segunda situao (quando temos os vetores
induo magntica criados por cada fio retilneo e tambm o
criado pela corrente i). Logo:

1 2 1 2
2 B B B B B B B + = + =
, , , , , , ,

( )
1 2
2
4 2
+

=
t
i i
i
R R

1 2
4
i i
i
+
=
t


QUESTO 20
Uma lua de massa m de um planeta distante, de massaM m > ,
descreve uma rbita elptica com semieixo maior a e semieixo
menor b , perfazendo um sistema de energia E . A lei das reas
de Kepler relaciona a velocidade v da lua no apogeu com sua
velocidade ' v no perigeu, isto , '( ) ( ) v a e v a e = + , em que e a
medida do centro ao foco da elipse. Nessas condies, podemos
afirmar que

a) / (2 ) E GMm a = .
b) / (2 ) E GMm b = .
c) / (2 ) E GMm e = .
d) / E GMm a b = + .
e) ' 2 / ( ) v GM a e = .
Resoluo Alternativa A

Como a massa M do planeta muito maior que a massa m da lua,
podemos considerar que o planeta encontra-se esttico no referencial
adotado.
Um desenho pode tornar mais fcil a resoluo do exerccio.
i
R
i
i
1
i
2
2R
2R
i
1
i
2


(19) 3251-1012 O ELITE RESOLVE ITA 2013 - FSICA

12

a
e
v
' v


Podemos agora tentar encontrar a energia E em funo da energia
potencial gravitacional e das energias cinticas nos pontos de apogeu
(ponto de maior distncia entre planeta e lua) e perigeu (ponto mais
prximo). As energias sero iguais, pois o sistema conservativo.
2 2
'
2 2
apogeu perigeu
m v GMm m v GMm
E
a e a e

= =
+


Como o enunciado nos diz que '( ) ( ) v a e v a e = + ento:
2
2
2 2
| | + | |
=
| |
+
\ . \ .
m v GMm m a e GMm
v
a e a e a e

Buscando isolar
2
2
m v | |
|
\ .
:
2
2
1 1
1
2
m a e
GMm v
a e a e a e
| |
+ | | | |
= |
| |
|
+
\ . \ .
\ .

( )
2
2 2 2
2 4
2
| |
| |
| =
|
|

\ .
\ .
e mv ae
GMm
a e
a e

( ) ( )
2
+
e
GMm
a e a e
( )
2
2
4
2
| |
|
=
|

\ .
mv ae
a e
| |
|
|
\ .

2
2 2
m v GMm a e
a e a
| |
=
|
+
\ .

Se substituirmos a energia cintica encontrada no apogeu ficamos
com:
2
GMm a e GMm
E
a e a a e
| |
=
|
+ +
\ .

1
2
GMm a e
E
a e a
| |
=
|
+
\ .

2
2 2
GMm a e a
E
a e a a
| |
=
|
+
\ .

2
GMm a e
E
a e a
+ | |
=
|
+
\ .

2
GMm
E
a
=

QUESTO 21
Considere as seguintes relaes fundamentais da dinmica
relativstica de uma partcula: a massa relativstica =
0
m m , o
momentum relativstico =
0
p m v e a energia relativstica
2
0
E m c = , em que
0
m a massa de repouso da partcula e
2 2
1 1 v c = o fator de Lorentz. Demonstre que
( )
2
2 2 2 2
0
E p c m c = e, com base nessa relao, discuta a
afirmao: Toda partcula com massa de repouso nula viaja com
a velocidade da luz c.
Resoluo
Inicialmente, observe que podemos reescrever o fator de Lorentz
como:

2 2 2 2 2
2 2
1 1
1
c
v c v c v
c c
= = =

,

definido para v c < .

Assim, para v c < , de acordo com as relaes dadas, temos que:

( ) ( )
( )
2
2
2 2 2 2 2 2 2 2 2 2
0 0 0
E p c m c m v c m c c v = = =
2
0
2 2
c
m
c v
=

( )
2
2 2 2
c c v
| |

|
|
\ .
( )
2
2 2 2 2
0 0
m c c m c = = ,

que o que queramos demonstrar.

Por outro lado, da energia relativstica, temos que:

2
0 0 2
E
E m c m
c
= =

Substituindo no momentum relativstico, vem que:

0 2
E
p m v v
c
= =

Voltando equao que acabamos de demonstrar:

( )
2
2 2
2
2 2 2 2 2 2 2 2
0 2 4
E E v
m c E p c E v c E c
c c
| |
= = =
|
\ .

( )
2
2
2 2
0 2
1
v
E m c
c
| |
=
|
\ .


A discusso a partir desse ponto tem que ser levada com certo
cuidado. Primeiramente, preciso observar que mesmo as
partculas com massa de repouso nula, como os ftons, tm
energias no nulas associadas a elas (segundo a Fsica Quntica,
energias da forma h f). Assim, a energia E sempre diferente de
zero. Segundo, o fator de Lorentz no fica bem definido quando
v = c, pois seria nulo o denominador da frao que o define.
Portanto, imaginamos que o propsito aqui seja pensar que,
fazendo a massa de repouso tender a zero (m
0
0), e a energia
E no sendo nula, segue que:

2
0 2
0 1 0
v
m v c
c


Isso permitiria afirmar que, de fato, partculas com massa de
repouso nula viajam com a velocidade da luz c.

QUESTO 22
Um recipiente inicialmente aberto para a atmosfera a
temperatura de 0 C. A seguir, o recipiente fechado e imerso
num banho trmico com gua em ebulio. Ao atingir o novo
equilbrio, observa-se o desnvel do mercrio indicado na escala
das colunas do manmetro. Construa um grfico P xT para os
dois estados do ar no interior do recipiente e o extrapole para
encontrar a temperatura
0
T quando a presso 0 P = , interpretando
fisicamente esse novo estado luz da teoria cintica dos gases.


cm cm
1 1
P T
-14
14
0
2 2
P T
-14
14
0



Resoluo
Devemos marcar no grficoP T os dois pontos referentes aos
estados, inicial e final. O estado inicial corresponde presso
atmosfrica 76 cmHg
i
P = e temperatura de 0 C
i
T = . O estado
final, correspondente temperatura de ebulio da
gua 100 C
f
T = , apresentou uma medida de presso superior em
28 cmHgcom relao ao estado inicial, ou seja, 104 cmHg
f
P = .



(19) 3251-1012 O ELITE RESOLVE ITA 2013 - FSICA

13

Colocando esses dois pontos no grfico podemos traar uma reta
e encontrar a temperatura
0
T onde 0 P = :
P (cmHg)
T (C)
0
100
76
104
T0

Esse valor pode ser encontrado pela equao da reta

104 76
76
100
P T
| |
= +
|
\ .
0,28. 76 P T = +
e fazendo 0 P = :
0
0 0,28. 76 T = +
0
76
271,4
0,28
T = ~

0
271,4 C T ~

Considerando o ar como um gs ideal e a poro de ar dentro do
manmetro desprezvel, perante o volume no recipiente, ele sofre
uma transformao aproximadamente isomtrica.
Assim, para a temperatura T em uma escala absoluta (por
exemplo, a escala Kelvin),
i
i
P P
T T
=
Ento, de acordo com essa expresso o grficoP T deve ser
uma reta
.
i
i
P
P T
T
| |
=
|
\ .

tal que a temperatura deve ser
0
0K T = para 0 P = . Isso significa
que a temperatura
0
0K T = corresponde ao zero absoluto, que de
acordo com a teoria cintica dos gases a temperatura mais baixa
que pode ser atingida. De acordo com essa teoria, a temperatura
uma grandeza relacionada energia cintica mdia das molculas
de um corpo, e o zero absoluto a temperatura correspondente a
um estado de energia cintica nula.
O exerccio mostra uma maneira de se determinar
experimentalmente a temperatura do zero absoluto.

QUESTO 23
Num plano horizontal x x y , um projtil de massa m lanado
com velocidadev , na direo de u com o eixo x , contra o centro
de massa de uma barra rgida, homognea de comprimentoL e
massaM , que se encontra inicialmente em repouso a uma
distnciaD de uma parede, conforme a figura. Aps uma primeira
coliso elstica com a barra, o projtil retrocede e colide
elasticamente com a parede. Desprezando qualquer atrito,
determine o intervalo de valores u para que ocorra uma segunda
coliso com a barra, e tambm o tempo decorrido entre esta e a
anterior na parede.



y
M
D
L/2
L/2
m
v

X


Resoluo
Inicialmente, iremos decompor a velocidade v
,
do projtil em uma
componente horizontal cos
x
v v = u e outra vertical sen
y
v v = u .
Este projtil estar sujeito a duas interaes:
i) coliso elstica com a barra rgida, e
ii) coliso elstica contra a parede esquerda.
Na primeira, devido ausncia de atrito, sua componente
y
v da
velocidade permanece inalterada, podendo, contudo, transferir
momento linear para a barra, fazendo com que esta adquira
movimento ao longo da direo x
,
.
Caso o projtil tenha velocidade de recuo negativa na direo x
,
,
ele poder ainda sofrer uma segunda coliso, agora contra a
parede. Nesta coliso, sua velocidade em y
,
continua inalterada,
entretanto a coliso inverte o sentido de seu movimento ao longo
de x
,
, fazendo o projtil se mover em direo barra novamente.
Denominaremos por:
u
,
: velocidade da barra aps a primeira coliso com o projtil;
v'
,
: velocidade do projtil aps a primeira coliso com a barra;
v''
,
: velocidade do projtil aps a coliso contra a parede.

Na primeira coliso, como a velocidade em y
,
no se altera, para
o projtil e a barra no adquire movimento nesta direo,
sen
y y
v v v ' = = u ,
0
y
u = .
Para as componentes horizontais, temos conservao da
quantidade de movimento e a restrio de a coliso ser
perfeitamente elstica (velocidade de afastamento igual de
aproximao), ou seja:
x x x
mv Mu mv' = + ,
x x x
v u v' = .
Resolvendo o sistema, encontramos:
- multiplicando a segunda equao por M e somando as duas
equaes:
cos
x
m M
v v
m M

' = u
+
,

- multiplicando a segunda equao por m e somando as duas
equaes:
2
cos
x
m
u v
M m
= u
+
.
Para que a velocidade de recuo em x
,
seja negativa, devemos ter
satisfeita a condio M m > . Estando este critrio atendido, na
segunda coliso, contra a parede, temos
cos
x x
M m
v v v
M m

'' ' = = u
+
,
sen
y y
v v v '' ' = = u .
Para que o projtil volte a se encontrar com a barra, aps um
intervalo de tempo t aps o primeiro encontro, ele deve atender
as seguintes condies:
a) regressar a distncia D at a parede, entre as colises,
b) progredir a distncia
x
D u + t , na direo x, at onde a
barra se localiza;
c) ter um deslocamento vertical h menor do que / 2 L .

Assim, o tempo de trajeto total ser:
2
x x
x x x x
D u u D D
v v v v
+ t t
t = + = +
' '' '' ''
(1)
2 2
cos
M m D m
M m v M m
+
t = + t
u

Isolando t :
2
3 cos
M m D
M m v
+
t =
u
.
Note que para haver soluo ( 0 t > ), devemos ter 3 M m > . Isto
mostra que o problema tem soluo para M nesta faixa de
valores, que satisfaz a j estipulada M m > . O deslocamento
vertical total ser dado por


(19) 3251-1012 O ELITE RESOLVE ITA 2013 - FSICA

14

2
sen
3 cos
y
M m D
h v v
M m v
+
= t = u
u
2 tan
3
M m
h D
M m
+
= u

.
Como queremos que 2 h L < ,
2 tan
3 2
M m L
D
M m
+
u <

3
tan
4
L M m
D M m

u <
+
.
Por construo, sabemos que 0 u > , logo
3
0 arctan
4
L M m
D M m
| |
s u <
|
+
\ .
.

Para calcular o tempo T de coliso entre o choque contra a
parede e o segundo encontro com a barra, consideramos apenas
a segunda contribuio de (1), ou seja,
x
x
D u
T
v
+ t
=
''

( )
( ) ( )
( )
( )
2 cos 2
cos cos 3 cos
D M m M m mv M m D
T
M m v M m M m v M m v
+ + u +
= +
u + u u

( )
( ) ( )
2 2
cos 3 cos
D M m m M m D
T
M m v M m M m v
+ +
= +
u u

( )
( )
4
1
cos 3
D M m m
T
M m v M m
+
| |
= +
|
u
\ .

( )
( )( )
2
cos 3
M m D
T
v M m M m
+
=
u
.
QUESTO 24
Dois radiotelescpios num mesmo plano com duas estrelas
operam como um interfermetro na freqncia de 2,1 GHz. As
estrelas so interdistantes de 5,0 L = anos-luz e situam-se a uma
distncia
7
2,5x10 D = anos-luz da Terra. Ver figura. Calcule a
separao mnima, d , entre os dois radiotelescpios necessria
para distinguir as estrelas. Sendo 1 u << em radianos, use a
aproximao tan sen u ~ u ~ u .

Resoluo
Considere inicialmente somente a luz de uma estrela, chegando em
ambos os radiotelescpios como ondas planas. Esse sistema funciona
exatamente como uma experincia de fenda dupla, onde os
radiotelescpios fazem o papel das fendas (captador da luz/ondas
planas). Portanto, se forma um padro de interferncia como mostrado
na figura abaixo.
D
d
a
N
y

Sabemos que as posies
N
y de mximos e mnimos de interferncia
(construtivas e destrutivas) so dadas por

2
N
a
y N
d

= ,

ondeN par/mpar para as franjas de interferncia
construtiva/destrutiva.

Considere agora a luz proveniente da segunda estrela, chegando aos
radiotelescpios a um ngulou . As franjas de interferncia da luz
proveniente dessa estrela sero deslocadas em um ngulou com
relao s franjas da primeira.


L
D
d
a
1
y

u


Como uma boa aproximao, podemos dizer que o interfermetro
conseguir distinguir as duas estrelas quando o mximo central da
segunda estiver exatamente sobre o primeiro mnimo da primeira
( 1 N = ):

1
2
y
a d

= . (1)

Da figura podemos ver que, pela aproximao deu pequeno


1
tan
y
a
u u = = ,

e ainda

tan
L
D
u u = = .

Dessa forma,

1
y L
D a
=
(1)

2
L
D d

=

2
D
d
L

= . (2)

O comprimento de onda pode ser encontrado pela relao

c f =
8
9
3.10 1
2,1.10 7
c
f
= = = m.

Ento, substituindo os valores na equao (2) temos

7
5
1 2,5.10
. 3,57.10
2 7 2.5
D
d
L
= = =

357km d =
QUESTO 25
Em atmosfera de ar calmo e densidade uniforme
a
d , um balo
aerosttico, inicialmente de densidade d , desce verticalmente
com acelerao constante de mduloa . A seguir, devido a uma
variao de massa e de volume, o balo passa a subir
verticalmente com acelerao de mesmo mdulo a . Determine a
variao relativa do volume em funo da variao relativa da
massa e das densidades
a
d e d .


(19) 3251-1012 O ELITE RESOLVE ITA 2013 - FSICA

15

Resoluo
Para o balo, da definio de densidade, temos:
Volume inicial:
0
0
m
V
d
=
Volume final:
F
F
F
m
V
d
=
Variao relativa de volume:
0
0 0
F
V V V
V V
A
=
0
0
0 0 0
0 0
1
F F
F F F
F
m m m
m
d d d m V d
d
m m m
V d m
d d d

A
= = =
0 0
1
F
F
dm V
V d m
A
= , como
0 F
m m m = + A , temos:
( )
0
0 0
1
F
d m m V
V d m
+ A A
=
0 0
1 1
F
V d m
V d m
| | | | A A
= +
| |
\ .\ .
(I)

Para a situao do balo em descida acelerada, com acelerao
de mdulo a, de acordo com a 2 Lei de Newton, temos:
0 0 0
P E m a =
0 0 0 a
m g d V g m a =
0
0
a
d V
g g a
m
= , como
0
0
1 V
m d
= , temos:
1
a
d
g a
d
| |
=
|
\ .
(II)
Para a situao do balo em subida acelerada, com acelerao de
mdulo a, de acordo com a 2 Lei de Newton, temos:
F F F
E P m a =
a F F F
d V g m g m a =
a F
F
d V
g g a
m
= , como
1
F
F F
V
m d
= , temos:
1
a
F
d
g a
d
| |
=
|
\ .
(III)
De (III) e (II) temos:
1 1
a a
F
d d
g g
d d
| |
| |
=
|
|
\ .
\ .

1 1
a a
F
d d
d d
=
2
a a
F
d d
d d
=
2
a a
F
d d d
d d

=
2
a
F
a
d d
d
d d

(IV)

Substituindo (IV) em (I), temos:
0 0
1 1
2
a
a
V d m
d d
V m
d d
| |
|
| | A A
|
= +
|
|
\ .
|

\ .

0 0
2
1 1
a
a
d d V m
V d m
| || | A A
= +
| |
\ .\ .

Logo, a variao volumtrica relativa :
0 0
2
1 1 1
a
V d m
V d m
| || | A A
= +
| |
\ .\ .
.


QUESTO 26
Um mol de um gs ideal sofre uma expanso adiabtica reversvel
de um estado inicial cuja presso P
i
e o volume V
i
para um
estado final em que a presso P
f
e o volume V
f
. Sabe-se que
= C
p
/C
v
o expoente de Poisson, em que C
p
e C
v
so os
respectivos calores molares a presso e a volume constantes.
Obtenha a expresso do trabalho realizado pelo gs em funo de
P
i
, V
i
, P
f
, V
f
e .
Resoluo
Seja f o nmero de graus de liberdade de cada molcula do gs, e
R a constante universal dos gases perfeitos. Temos que:

2
2
2
p
v
f
C R
f
C R
+ | |
=
|

\ .

,

de onde segue que:

2 2 2 1
1 1
2 1
p
v
C
f f
C f f f
+
= = = + = =



Da Primeira Lei da Termodinmica, temos que:

Q U = t + A

Sendo a transformao adiabtica, o calor absorvido pelo gs
nulo.
Assim:

0 Q U = t = A

A variao de energia interna do gs dada por:

( )
2
f f i i
f
U P V P V A = ,

Portanto:

( )
1
1
f f i i
P V P V t =

( )
1
1
f f i i
P V P V t =



QUESTO 27
Um dispositivo usado para determinar a distribuio de
velocidades de um gs. Em 0 t = , com os orifcios ' O e O
alinhados no eixo z , molculas ejetadas de ' O , aps passar por
um colimador, penetram no orifcio O do tambor de raio interno
R , que gira com velocidade angular constante e. Considere, por
simplificao, que neste instante inicial ( 0 t = ) as molculas em
movimento encontram-se agrupadas em torno do centro do orifcio
O . Enquanto o tambor gira, conforme mostra a figura, tais
molculas movem-se horizontalmente no interior deste ao longo
da direo do eixo z , cada qual com sua prpria velocidade,
sendo paulatinamente depositadas na superfcie interna do tambor
no final de seus percursos. Nestas condies, obtenha em funo
do ngulo u a expresso para
min
v v , em que v a velocidade
da molcula depositada correspondente ao giro u do tambor e
min
v a menor velocidade possvel para que as molculas sejam
depositadas durante a primeira volta deste.
Colimador
e
R
u
z
' O
O

Resoluo
O tempo que a molcula leva para atravessar o interior do tambor
ao longo do dimetro dado por
2R
t
v
= .


(19) 3251-1012 O ELITE RESOLVE ITA 2013 - FSICA

16

Durante este intervalo de tempo o tambor efetua um giro u dado
por
t u = e .
Eliminando o tempo t das equaes acima, obtemos
2 R
v
e
=
u
.
As partculas mais lentas que penetram no tambor se depositam
aps uma revoluo completa dele, ou seja, aps 2 u = t , de modo
que
min
R
v
e
=
t
.
Com isso, temos
min
2 R R
v v
e e
=
u t
min
2
1
R
v v
e t | |
=
|
t u
\ .
.
Ou equivalentemente, em unidades de
min
v ,
min min
2
1 v v v
t | |
=
|
u
\ .


QUESTO 28
O experimento mostrado na figura foi montado para elevar a
temperatura de certo lquido no menor tempo possvel,
dispendendo uma quantidade de calor Q . Na figura, G um
gerador de fora eletromotriz c , com resistncia eltrica internar ,
e R a resistncia externa submersa no lquido. Desconsiderando
trocas de calor entre o lquido e o meio externo,
a) Determine o valor de R e da correntei em funo de c e da
potncia eltrica P fornecida pelo gerador nas condies
impostas.
b) Represente graficamente a equao caracterstica do gerador,
ou seja, a diferena de potencial U em funo da intensidade da
corrente eltricai .
c) Determine o intervalo de tempo transcorrido durante o
aquecimento em funo de Q , i e c .
R
G
r
c


Resoluo
Como se deseja realizar um aquecimento no menor tempo, devemos
despender na resistncia R a maior potncia possvel. Esta condio
estabelece uma relao entre r e R .
A resistncia equivalente do circuito R r + , desse modo a corrente
i dada por
i
R r
c
=
+
. (1)
A potncia P dissipada em R ser
( )
2
P R i i r i = = c .
Esta expresso quadrtica em i e ser mxima (
mx
P ) quando a
corrente
mx
i for a mdia das razes, conforme ilustra o grfico a
seguir, ou seja:
P
i
i
2 i
1
1 2
2
i i +

P
mx

1
2
0
0
i
P
i
r
=

=
c
=


2
mx
i
r
c
=
Substituindo (1) na expresso acima, obtemos:
2
2
R r r
r R r
c c
= + =
+
R r = .
Com isso,
2
2
2
2 4
P Ri R P
R R
c c | |
= = =
|
\ .
(2)

a) Queremos expressar R e i em termos de P e c . Para R
reescrevemos a equao (2) na forma

2
4
R
P
c
= ,

e para i , reescrevemos a equao (1) substituindo o resultado de (2):

2
4
2 2
P
i
R
c c
= =
c
2P
i =
c
. (3)

b) A diferena de potencial (d.d.p.) U nas extremidades do
gerador

U r i = c .

Assim, o grfico dessa d.d.p. em funo da corrente i

U
i
c
R
c

0


c) A quantidade de calor Q que ser fornecida pela potncia P
dissipada na resistncia, dada pela expresso (3):

2
i
P
c
= .

Dessa forma, o tempo t necessrio ser
2
i t
Q P t
c
= =
2Q
t
i
=
c
.

QUESTO 29
Duas placas condutoras de raio R e separadas por uma distncia
<< d R so polarizadas com uma diferena de potencial V por
meio de uma bateria. Suponha sejam uniformes a densidade
superficial de carga nas placas e o campo eltrico gerado no
vcuo entre elas. Um pequeno disco fino, condutor de massa m e
raio r , colocado no centro da placa inferior. Com o sistema sob
a ao da gravidade g , determine, em funo dos parmetros
dados, a diferena de potencial mnima fornecida pela bateria para
que o disco se desloque ao longo do campo eltrico na direo da
placa superior.
Resoluo
A figura abaixo sintetiza algumas das idias do problema.


r
R
d
V
r
R
d
V
E
,



Existem duas placas condutoras separadas a uma distncia d e sob
diferena de potencial V . Isto gera entre as placas um campo eltrico,


(19) 3251-1012 O ELITE RESOLVE ITA 2013 - FSICA

17

que consideraremos uniforme como o enunciado sugere, dirigido da
placa positiva para a negativa e de intensidade
V
E
d
= .
A partir da lei de Gauss, podemos mostrar que, na superfcie de
qualquer uma das placas condutoras, forma-se uma densidade de
carga dada por
0
V
d
c
o = ,
onde
0
c a permissividade eltrica do vcuo, meio que ocupa o
espao entre as placas. Aqui a densidade positiva diz respeito placa
inferior, e a negativa, placa superior. Ao encostarmos o fino disco
condutor sobre a placa inferior, este tambm ir adquirir a mesma
densidade superficial de cargas o , de modo a possuir carga total
2 2 0
V
Q r r
d
c
= t o = t ;
em que
2
r t a rea do disco.
Para que o disco flutue em direo placa superior, deve atuar no
disco uma fora eltrica que supere seu peso, ou seja
Q E m g > .
Assim, resolvendo esta inequao para o potencial V :
2
2 2 0
2
0
V V d m g
r m g V
d d r
c
t > >
tc

0
d m g
V
r

>
tc
.

QUESTO 30
Um prton em repouso abandonado do eletrodo positivo de um
capacitor de placas paralelas submetidas a uma diferena de
potencial c = 1000 V e espaadas entre si de d = 1 mm, conforme
a figura. A seguir, ele passa atravs de um pequeno orifcio no
segundo eletrodo para uma regio de campo magntico uniforme
de mdulo B = 1,0 T. Faa um grfico da energia cintica do
prton em funo do comprimento de sua trajetria at o instante
em que a sua velocidade torna-se paralela s placas do capacitor.
Apresente detalhadamente seus clculos.


Resoluo
A energia cintica K do prton abandonado aumentar sempre
que alguma fora realizar sobre ele trabalho. Neste problema h
dois ambientes, um no qual o prton est sob ao de uma fora
eltrica
E
F
,
, promovida pela diferena de potencial entre as placas
do capacitor, e outro no qual atua sobre ele uma fora magntica
M
F
,
.

A fora eltrica proporcional carga q + do prton e ao campo
/ E d = c entre as placas:
E
q
F q E
d
c
= = .
O trabalho realizado por esta fora proporcional distncia x
percorrida pelo prton entre as placas. De acordo com o teorema
do trabalho-energia cintica, temos que:

( ) ( ) 0 , 0
RES
q
K x x K x x d
d
c
t = = s s ,

atingindo valor mximo igual a:

( )
19 16
1,6 10 1000 1,6 10 J K d q

= c = = .

Para este valor mximo, quando o prton abandona o ambiente
onde h a fora eltrica, sua energia cintica vale q c , e sua
velocidade pode ser calculada por:

2
2
2
m v q
q v
m
c
= c = .

A fora magntica, por sua vez, no realiza trabalho sobre o
prton. Toda fora magntica perpendicular velocidade, e atua
alterando a direo do movimento apenas. Neste caso, ela far o
prton seguir uma trajetria circular, funcionando como uma
resultante centrpeta
C
R
,
. Ou seja,

( ) , K x q x d = c >

Para construir o grfico exigido, j temos o comportamento da
funo ( ) K x para qualquer valor da distncia percorrida pelo
prton. Precisamos agora determinar o ponto final
f
x do grfico,
quando sua velocidade perpendicular placa do capacitor.
Na figura abaixo podemos ver que sua velocidade perpendicular
aps ele se deslocar por um quarto de arco de circunferncia, ou
seja, aps percorrer uma distncia
2
R
t
.


O raio da trajetria circular pode ser encontrado igualando a fora
magntica resultante centrpeta:

2
sen90
M C
m v
F R q v B
R

= =
2
2 m v m
R R
q B q B
c
= =

.

Com isso, a distncia final
f
x ser igual a:

2
2
2 2
f
m
x d R d
q B
t t c
= + = +

.

Substituindo os valores dados no enunciado e no cabealho da
prova, obtemos:

2 27
3 3
19 2
1000 1,7 10
10 8,24 10 m 8,24 mm
2 1,6 10 1,0
f
x

t
= + ~ =



Assim, o grfico pedido :
x (mm)
K(x) (10
16
J )
0
1,6
1,0 8,24

B
q
d
c


(19) 3251-1012 O ELITE RESOLVE ITA 2013 FSICA

18






Equipe desta resoluo



Fsica
Claiton Pimentel de Oliveira
Luiz Salles de Carvalho
Matheus Veronez
Vagner Figueira de Faria
Vincio Meron Poltronieri


Reviso
Danilo J os de Lima
Edson Vilela Gadbem
Eliel Barbosa da Silva
Fabiano Gonalves Lopes


Digitao, Diagramao e Publicao
Ana Luiza Brunetti
Isabela Porto Ren
Lucas dos Santos Vargette

Você também pode gostar